Be careful when we use the polar method!

Поделиться
HTML-код
  • Опубликовано: 27 окт 2024

Комментарии • 19

  • @e.n85784
    @e.n85784 3 месяца назад +5

    Finally I will be here from the past

    • @mohannad_139
      @mohannad_139 2 месяца назад +1

      How did you write this comment 2 weeks ago?

    • @e.n85784
      @e.n85784 2 месяца назад

      @@mohannad_139 this video was unlisted, and it was hyperlinked from another video doing some simpler limits in polar form.

  • @kirbo722
    @kirbo722 2 месяца назад +1

    The question doesn't seem to change because both x and y approach 0, but the real difference is the x³ on the top.
    Along y=0:
    1st case: lim (x³/(x²+0)) = lim (x) = 0
    2nd case: lim (x³/(x⁴+0)) = lim (1/x) => DNE
    :)

  • @markosth09
    @markosth09 3 месяца назад

    Great video

  • @eswyatt
    @eswyatt 2 месяца назад +1

    There's another textbook favorite where the limit approaches zero along every path except along y = Xsquared, where it approaches 1. How do you know which paths to check?

    • @xinpingdonohoe3978
      @xinpingdonohoe3978 2 месяца назад

      What's the limit in question? If we can see the form, we may be able to judge.

    • @bprpcalculusbasics
      @bprpcalculusbasics  2 месяца назад +1

      I have a video on "how to show nonexistence" coming soon!

  • @GlitchMaths
    @GlitchMaths 3 месяца назад +3

    I think you haven
    t listed it yet

  • @erroraftererror8329
    @erroraftererror8329 2 месяца назад +2

    Do you have a question like this such that the limit would equal one? That’d be cool to see.

    • @xinpingdonohoe3978
      @xinpingdonohoe3978 2 месяца назад

      (x²+y³)/(x²+y³)

    • @erroraftererror8329
      @erroraftererror8329 2 месяца назад

      @@xinpingdonohoe3978 Well, you did answer my question, but I was hoping for something more exciting than that.

    • @pedropiata648
      @pedropiata648 2 месяца назад

      ​@@xinpingdonohoe3978that one is sooo hard

    • @bprpcalculusbasics
      @bprpcalculusbasics  2 месяца назад

      Have you checked out my "6 methods of evaluating the limit of a multivariable function"? The link is in the description and hopefully it answers your question.

    • @erroraftererror8329
      @erroraftererror8329 2 месяца назад

      @@bprpcalculusbasicsI was talking about a polar limit question that would approach 1. The example you have in the video approaches 0.

  • @dsx0164
    @dsx0164 2 месяца назад

    Ok.. but how can I see this paths in general case

    • @yurfwendforju
      @yurfwendforju 2 месяца назад +1

      This video makes me feel like when I find a bug in a video game and it gets patches :'(

  • @windowsxpmemesandstufflol
    @windowsxpmemesandstufflol 3 месяца назад

    wow